possible triangle inequality
by sunshine_12, Mar 30, 2025, 2:12 PM
a, b, c are real numbers
|a| + |b| + |c| − |a + b| − |b + c| − |c + a| + |a + b + c| ≥ 0
hey everyone, so I came across this inequality, and I did make some progress:
Let (a+b), (b+c), (c+a) be three sums T1, T2 and T3. As there are 3 sums, but they can be of only 2 signs, by pigeon hole principle, atleast 2 of the 3 sums must be of the same sign.
But I'm getting stuck on the case work. Can anyone help?
Thnx a lot
|a| + |b| + |c| − |a + b| − |b + c| − |c + a| + |a + b + c| ≥ 0
hey everyone, so I came across this inequality, and I did make some progress:
Let (a+b), (b+c), (c+a) be three sums T1, T2 and T3. As there are 3 sums, but they can be of only 2 signs, by pigeon hole principle, atleast 2 of the 3 sums must be of the same sign.
But I'm getting stuck on the case work. Can anyone help?
Thnx a lot
This post has been edited 2 times. Last edited by sunshine_12, Yesterday at 2:41 PM
Polynomials and their shift with all real roots and in common
by Assassino9931, Mar 30, 2025, 1:12 PM
We call two non-constant polynomials friendly if each of them has only real roots, and every root of one polynomial is also a root of the other. For two friendly polynomials
and a constant
, it is given that
and
are also friendly polynomials. Prove that
.





This post has been edited 1 time. Last edited by Assassino9931, Yesterday at 1:13 PM
Thanks u!
by Ruji2018252, Mar 30, 2025, 11:07 AM
VERY HARD MATH PROBLEM!
by slimshadyyy.3.60, Mar 29, 2025, 10:49 PM
Let a ≥b ≥c ≥0 be real numbers such that a^2 +b^2 +c^2 +abc = 4. Prove that
a+b+c+(√a−√c)^2 ≥3.
a+b+c+(√a−√c)^2 ≥3.
Harmonic Series and Infinite Sequences
by steven_zhang123, Mar 29, 2025, 12:41 AM
Let
and
be two infinite sequences of integers. Prove that there exists an infinite sequence of integers
such that for any positive integer
, the following holds:
![\[
\sum_{k|n} k \cdot z_k^{\frac{n}{k}} = \left( \sum_{k|n} k \cdot x_k^{\frac{n}{k}} \right) \cdot \left( \sum_{k|n} k \cdot y_k^{\frac{n}{k}} \right).
\]](//latex.artofproblemsolving.com/2/5/9/259ee32c89befe7a3ee2797e0d5b1df1c3066cf6.png)




![\[
\sum_{k|n} k \cdot z_k^{\frac{n}{k}} = \left( \sum_{k|n} k \cdot x_k^{\frac{n}{k}} \right) \cdot \left( \sum_{k|n} k \cdot y_k^{\frac{n}{k}} \right).
\]](http://latex.artofproblemsolving.com/2/5/9/259ee32c89befe7a3ee2797e0d5b1df1c3066cf6.png)
Inspired by old results
by sqing, Mar 27, 2025, 12:35 PM
Let
and
Prove that
Let
and
Prove that







This post has been edited 1 time. Last edited by sqing, Mar 27, 2025, 12:44 PM
A cute FE
by Aritra12, Mar 23, 2021, 3:17 PM
Hope so not prediscovered
Find all functions
such that for all reals 
Proposed by Aritra12, India
Click to reveal hidden text
Its simple but yet cute acc to me
Find all functions



Click to reveal hidden text
i will post the solution after some time
edit
solution
First,
and
imply that either
or
.
In the first case,
implies that
. In the second case,
also implies that
. Thus,
in every case.
implies that for any
, either
or
.
If
, then
is injective.
and
imply that
for all
. From here it’s easy to get
.
Else, there is
such that
, then comparing
and
gives
for all such
. Plugging in
then gives
and thus
. Hence
which means that
.
Since for any
, either
or
, we have
for all
. However, if
for some
, then
which is impossible. Thus
in this case.
edit
solution
First,




In the first case,









If







Else, there is











Since for any









Its simple but yet cute acc to me
This post has been edited 6 times. Last edited by Aritra12, Jun 13, 2021, 12:47 PM
x is rational implies y is rational
by pohoatza, Jun 28, 2007, 7:24 PM
For
let
be the number whose
-th digit after the decimal point is the
-th digit after the decimal point of
. Show that if
is rational then so is
.
Proposed by J.P. Grossman, Canada







Proposed by J.P. Grossman, Canada
USAMO 1995
by paul_mathematics, Dec 31, 2004, 1:01 PM
Given a nonisosceles, nonright triangle ABC, let O denote the center of its circumscribed circle, and let
,
, and
be the midpoints of sides BC, CA, and AB, respectively. Point
is located on the ray
so that
is similar to
. Points
and
on rays
and
, respectively, are defined similarly. Prove that lines
,
, and
are concurrent, i.e. these three lines intersect at a point.














Archives





















Shouts
Submit
686 shouts
Contributors
aaaaarush • addyc • agoel2010 • aguathegreat • Alvie567 • Aminecraftbear • Beast_Academic_Emma • Bob_Smart • CC_chloe • chaihanrui • Cokevending56 • dbnl • DU4532 • em10 • enya_yurself • ericwzzhang • erringbubble • gamma_pi • goldenuni678 • Hestu_the_Bestu • hiloo • ICED • IcyBoba • IshiDe • ishipoopman • joshualiu315 • juicetin.kim • jxli.94 • linny.t • madeleinelee • math_sp • MrThinker • neha.s • Obuy • poplint0507 • potato_potato_potato • pupitrethebean • RandomeMids • SapphireDolphin9 • Seraph58750 • teasang • Technodoggo • ujulee • vanstheshoe • VRobot • WaterBeast
Tags
About Owner
- Posts: 4
- Joined: Jan 19, 2023
Blog Stats
- Blog created: Mar 31, 2023
- Total entries: 677
- Total visits: 24599
- Total comments: 2440
Search Blog